Joint probability density function problem

Click For Summary
The discussion centers on proving that P(X<Y<Z)=1/6 for the joint probability density function f(x,y,z) = h(x)h(y)h(z). Participants express confusion about the proof process, especially since it is considered a review topic. They explore the concept of defining subsets based on permutations of the random variables and question the disjoint nature and shape of these subsets. Additionally, they inquire about the implications of functions invariant under parameter swapping and how this relates to the original probability problem. Overall, the conversation emphasizes the need for clarity on joint probability distributions and their properties.
braindead101
Messages
158
Reaction score
0
Suppose that h is the probability density function of a continuous random variable.
Let the joint probability density function of X, Y, and Z be
f(x,y,z) = h(x)h(y)h(z) , x,y,zER

Prove that P(X<Y<Z)=1/6


I don't know how to do this at all. This is suppose to be review since this is a continuation course, but I didn't take the previous course

Any help would be greatly appreciated
 
Physics news on Phys.org
For each permutation \sigma\in S_n you can define a following subset of \mathbb{R}^n.

<br /> X_{\sigma} := \{x\in\mathbb{R}^n\;|\; x_{\sigma(1)} \leq x_{\sigma(2)} \leq \cdots \leq x_{\sigma(n)}\}<br />

Some relevant questions: Are these subsets (with different \sigma) mostly/essentially disjoint? In what sense they are the same shape? How many of these subsets are there? What is the union \bigcup_{\sigma\in S_n} X_{\sigma}? If a function f:\mathbb{R}^n\to\mathbb{R} is invariant under a swapping of parameters like this

<br /> f(x_1,\ldots, x_n) = f(x_1,\ldots, x_{i-1},x_j, x_{i+1},\ldots x_{j-1}, x_i, x_{j+1},\ldots, x_n),<br />

what information does the restriction of f|_{X_{\sigma}} contain?

This is all related to your problem.
 
Last edited:
Question: A clock's minute hand has length 4 and its hour hand has length 3. What is the distance between the tips at the moment when it is increasing most rapidly?(Putnam Exam Question) Answer: Making assumption that both the hands moves at constant angular velocities, the answer is ## \sqrt{7} .## But don't you think this assumption is somewhat doubtful and wrong?

Similar threads

  • · Replies 19 ·
Replies
19
Views
4K
Replies
2
Views
2K
  • · Replies 3 ·
Replies
3
Views
2K
Replies
6
Views
2K
Replies
7
Views
1K
  • · Replies 4 ·
Replies
4
Views
2K
  • · Replies 11 ·
Replies
11
Views
2K
  • · Replies 6 ·
Replies
6
Views
2K
  • · Replies 5 ·
Replies
5
Views
2K
  • · Replies 4 ·
Replies
4
Views
2K